cr doubt

This topic has expert replies
Senior | Next Rank: 100 Posts
Posts: 50
Joined: Fri Jan 28, 2011 1:08 pm

cr doubt

by gmatapril » Fri Apr 08, 2011 4:34 am
When people evade income taxes by not declaring taxable income,a vicious cycle results。Tax evasion forces lawmakers to raise income tax rates,which causes the tax burden on non-evading taxpayers to become heavier This,in turn,encourages even more taxpayers to evade income taxes by hiding taxable income.
The vicious cycle described above could not result unless which of the following were true?
(A) An increase in tax rates tends to function as an incentive for taxpayers to try to increase their pretax incomes.
(B) Some methods for detecting tax evaders,and thus recovering some tax revenue lost through evasion,bring in more than they cost.but their success rate varies from year to yean
(C) When lawmakers establish income tax rates in order to generate a certain level of revenue,they do not allow adequately for revenue that will be lost through evasion.
(D) No one who routinely hides some taxable income can be induced by a lowering of tax rates to stop hiding such income unless fines for evaders are raised at the same time.
(E) Taxpayers do not differ from each other with respect to the rate of taxation that will cause them to evade taxes.


In this q, I am not sure as to whether we want to show whether vicious cycle should continue or whether vicious cycle should break..May be a silly doubt... But the OA seems to be on the lines that the vicious cycle should continue and question states that if which of the options were true, then vicious cycle could not result..
OA is C

User avatar
Legendary Member
Posts: 1101
Joined: Fri Jan 28, 2011 7:26 am
Thanked: 47 times
Followed by:13 members
GMAT Score:640

by HSPA » Fri Apr 08, 2011 4:49 am
Opinion: technique that will stop growth of tax evaders, modify existing evaders. Let me see choices
a) no one said that salries are increasing
b) only half the sentence is good, second half corrupted whole story
c) No one cares about evaders
d) lowering tax rates can modify evaders but it is opposite
e) waste of an option..

Nothing matched my opinion... Can I have OA and OE...
First take: 640 (50M, 27V) - RC needs 300% improvement
Second take: coming soon..
Regards,
HSPA.

User avatar
Legendary Member
Posts: 1101
Joined: Fri Jan 28, 2011 7:26 am
Thanked: 47 times
Followed by:13 members
GMAT Score:640

by HSPA » Fri Apr 08, 2011 4:51 am
I have seen OA... what is this 'best of the worst option' selection??

Did not like this question.. OE is mandatory.. there must be some serious explanation for this..
tough one for me
First take: 640 (50M, 27V) - RC needs 300% improvement
Second take: coming soon..
Regards,
HSPA.

Junior | Next Rank: 30 Posts
Posts: 12
Joined: Thu Jan 13, 2011 7:08 pm

by vjsharma25 » Fri Apr 08, 2011 5:00 am
HSPA wrote:I have seen OA... what is this 'best of the worst option' selection??

Did not like this question.. OE is mandatory.. there must be some serious explanation for this..
tough one for me
Its from OG(version I don't remember).Its a bit tricky,I agree.

Junior | Next Rank: 30 Posts
Posts: 12
Joined: Thu Jan 13, 2011 7:08 pm

by vjsharma25 » Fri Apr 08, 2011 5:00 am
HSPA wrote:I have seen OA... what is this 'best of the worst option' selection??

Did not like this question.. OE is mandatory.. there must be some serious explanation for this..
tough one for me
Its from OG(version I don't remember).Its a bit tricky,I agree.

Junior | Next Rank: 30 Posts
Posts: 12
Joined: Thu Jan 13, 2011 7:08 pm

by vjsharma25 » Fri Apr 08, 2011 5:01 am
Its from OG(version I don't remember).Its a bit tricky,I agree.

Legendary Member
Posts: 857
Joined: Wed Aug 25, 2010 1:36 am
Thanked: 56 times
Followed by:15 members

by AIM GMAT » Fri Apr 08, 2011 5:27 am
vjsharma25 wrote:Its from OG(version I don't remember).Its a bit tricky,I agree.
Its from OG 12 .
Thanks & Regards,
AIM GMAT

Legendary Member
Posts: 586
Joined: Tue Jan 19, 2010 4:38 am
Thanked: 31 times
Followed by:5 members
GMAT Score:730

by rohu27 » Fri Apr 08, 2011 9:13 am
let me try explaining: (apologies if it confuses you more, simply ignore it :D)
important thing here is question stem is wordy and confusing so first udnerstand what it asks for.

The vicious cycle described above could not result unless which of the following were true?
it asks for which of the following(if true) the cycle described above does not occur, the cycle will not occur when govt does not raise income tax burden on non-evading tax payers OR may be if there is a perfect way where in we can catch hold of all culprits, so there is no need for govt to raise tax.

With this in mind look at options. only B and C get through. both offer reasons if true the cycle may not occur.
but option B is not fool proof, it says the success varies from year to year, so we do not know that using this the cycle could be stopped.

option C-When lawmakers establish income tax rates in order to generate a certain level of revenue,they do not allow adequately for revenue that will be lost through evasion.
its a wordy option. all it says is govt does not decide upon a revenue generated via income tax keeping in mind that some percentage of people will evade it, so we got to collect more from the non-eavading ones. doesnt work out that way. they just have a set revenue, they will collect that irrespective of who evades or not.
so if the above is true, the govt will not raise tax when others evade it (govt deosnt try to compensate the evaders money by the non-evaders). so vicious cycle discussed will not occur.